You are on page 1of 5

100

Proforma Income Statement


January February March Total
Sales 263,500 186,000 217,000 666,500
Cost of goods sold 139,400 98,400 126,000 363,800
Gross profit 124,100 87,600 91,000 302,700
Selling expenses, 20% 52,700 37,200 43,400 133,300
Operating income 71,400 50,400 47,600 169,400
Interest expense 2,667 2,667 2,666 8,000
Income before tax 68,733 47,733 44,934 161,400
Income tax, 40% 27,493 19,093 17,974 64,560
Net income 41,240 28,640 26,960 96,840
99
Current unit cost per 1,000
Material P 52
Labor 20
Overhead 10
Total P 82

Effective January 1, 2007, the price of materials will be raised to P60. The unit cost for 2007
production will be P90. Since the sales of January and February come from December production,
only the March sales will have cost of P90 per thousand.

January and February cost of goods sold (1,700 + 1,200) x P82 P237,800
March 1,400 x P90 126,000
Cost of goods sold (first quarter) P363,800

98
Budgeted March sales 12,000
Add: Ending inventory units 15,860
Total units required 27,860
Less: Beginning inventory units 15,600
Budgeted purchases in units, March 12,260

97
Billings of December 31:
Collections with 3% discount P3,630,000 x 0.6 x 0.97 P2,112,660
Collections end of January P3,630,000 x 0.25 907,500
Billings of November 30: P3,540,000 x 0.09 318,600
Total collections P3,338,760

96
Payments for:
February purchases 54% x P2,436,000 P1,315,440
January purchases 46% x P2,250,000 1,035,000
Total payments for purchases P2,350,440
Selling, general and administrative expenses:
February: [(P3,420,000 x 0.15) – P20,000]0.54 266,220
January: [(P3,570,000 x 0.15) – P20,000]0.46 237,130
Total cash disbursements P2,853,790
95
January February
Budgeted sales 11,900 11,400
Add: Ending inventory (130%) 14,820 15,600
Total 26,720 27,000
Less: Beginning inventory 15,470 14,820
Budgeted purchases (units) 11,250 12,180
Unit purchase price 200 200
Budgeted peso purchases P2,250,000 P2,436,000

Budgeted inventories:
December 31 130% x 11,900 15,470
January 31 130% x 11,400 14,820
February 28 130% x 12,000 15,600
March 31 130% x 12,200 15,860
94
Beginning Cash P 20,000
Add: Cash collected on June's sales (P300,000 x .8 x .98) 235,200
Cash collected on May's sales ((P300,000/1.25) x .2) 48,000 283,200
Total P303,200
Less: Cash paid on June's purchases (P240,000 x .6 x .99) 142,560
Cash paid on May's purchases (P200,000 x .4) 80,000 222,560
Ending cash balance P80,640

93
November costs (P1,952,000 – P288,000) x 0.75 P1,248,000
October costs (P1,568,000 – P288,000) x 0.25) 320,000
Total disbursements P1,568,000
75
(C) For this calculation, the weighted-average cost of capital is based on the 16% cost of new
common stock and the 10% cost of debt. Retained earnings will not be considered because the
amount available has been exhausted. Thus, the weighted-average of any additional capital
required will be 13.6% [(60% x 16% cost of new equity) + (40% x 10% cost of new debt)].
Answer (A) is incorrect because 10% is the cost of debt capital. Answer (B) is incorrect because
12.74% is the weighted-average cost of capital calculated for a $7 million budget. Answer (D) is
incorrect because 16% is the cost of new common stock.

74
DISCUSSION: (B) To maintain a capital structure of 40% debt and 60% equity, the $7 million total must
consist of $2.8 million of debt and $4.2 million of equity. The equity will consist of $3 million of retained
earnings and $1.2 million of new stock. The weighted-average cost of the three sources of new capital
is determined as follows:
$3,000,000  $7,000,000 x 14% = 6.00%
$1,200,000  $7,000,000 x 16% = 2.74%
$2,800,000  $7,000,000 x 10% = 4.00%
12.74%
Answer (A) is incorrect because 11/14% assumes a tax adjustment for the cost of debt, but the 10% rate
is an after-tax amount. Answer (C) is incorrect because 13.6% assumes the equity consists solely of
new common stock. Answer (D) is incorrect because 16% is the cost of new common stock.

73
Answer (C) is correct. The current optimal capital structure is 40% debt and 60% equity. The $3 million
to be retained from earnings in the coming year represents the equity portion of the maximum new capital
outlay. To retain the optimal capital structure, $2 million of debt must be added to the $3 million of retained
earnings. Hence, the maximum capital expansion is $5 million.
Answer (A) is incorrect because $2 million is the amount of debt that must be added to maintain the
optimal structure. Answer (B) is incorrect because $3 million is the amount of earnings retained. Answer
(D) is incorrect because the amount of $5 million can be calculated.
72
(C) is correct. The company will receive only 80% of the $60 market price, or $48. Consequently, the dividend
yield is 6.25% ($3 ÷ $48). Adding the 10% growth rate produces a cost of new equity capital of 16.25%.
Answer (A) is incorrect because 6.25% ignores the dividend growth rate. Answer (B) is incorrect because
15% ignores the flotation costs. Answer (D) is incorrect because 10% is the dividend growth rate.

71
Answer (D) is correct. The cost of internal equity capital equals the dividend yield (dividends per share ÷
market price) plus the dividend growth rate. Dividing the $3 dividend by the $60 market price results in a
yield of 5%. Adding the 10% dividend growth rate produces a cost of 15% for retained earnings. No
adjustment is made for taxes because dividends are not tax deductible.
Answer (A) is incorrect because 5% is the dividend yield; the growth rate is ignored. Answer (B) is
incorrect because 9% would be the after-tax cost if dividends were deductible. Answer (C) is incorrect
because 10% is the dividend growth rate; it ignores the dividend yield.

70
Answer (B) is correct. Because the bonds are issued at their face value, the pretax effective rate is 10%.
However, interest is deductible for tax purposes, so the government absorbs 40% of the cost, leaving a
6% after-tax cost.
Answer (A) is incorrect because 4% assumes a 60% tax rate. Answer (C) is incorrect because 10% is
the before-tax rate. Answer (D) is incorrect because the after-tax cost will be less than the effective
before-tax rate.

69
(A) is correct. The cost of the bonds is given as 4.8%. The cost of the preferred stock is 8% ($8 dividend ÷
$100), the cost of new common stock is 7.6% ($7 dividend ÷ $92 proceeds), and the cost of the retained
earnings is 7% ($7 dividend ÷ $100 market price). These four costs are then weighted by the preferred
capital structure ratios, a process that requires subdividing the common stock portion into retained
earnings of $100,000 (10% of capital) and new common stock of $400,000 (40% of capital):
30% x 4.8% = 1.44%
20% x 8.0% = 1.60%
40% x 7.6% = 3.04%
10% x 7.0% = .70%
Total 6.78%
Rounding to the nearest tenth results in the correct answer of 6.8%.
Answer (B) is incorrect because 4.8% is the cost of the long-term debt. Answer (C) is incorrect because
6.5% would be correct only if the equity capital were obtained totally from retained earnings. Because
only $100,000 of retained earnings is available, the remainder of equity capital must come from sales of
new stock. Answer (D) is incorrect because 27.4% represents the unweighted total of each of the four
elements of cost.

68
Answer (C) is correct. Williams' preferred capital structure is 50% common stock. However, $100,000 of retained
earnings (50% of the required $200,000 of capital) will be used before any common stock is issued. Thus, the
weighted-average cost of capital will be determined based on the respective costs of the bonds, preferred stock,
and retained earnings. The cost of the bonds is given as 4.8%, the cost of the preferred stock is 8%, and the cost
of the retained earnings is 7% ($7 dividend ÷ $100 market price of the common stock). These three costs are then
weighted by the preferred capital structure ratios:
30% x 4.8% = 1.44%
20% x 8.0% = 1.60%
50% x 7.0% = 3.50%
Total 6.54%
Rounding to the nearest tenth produces the correct answer of 6.5%.
Answer (A) is incorrect because 19.8% represents the unweighted sum of each of the three elements of cost.
Answer (B) is incorrect because 4.8% is the cost of the long-term debt. All funding will not be obtained from debt
because the firm wants to maintain a capital structure in which debt represents only 30% of the total capital.
Answer (D) is incorrect because the 6.8% figure can only be obtained if new stock is sold. New stock will not be
sold because the retained earnings can be used at a lower cost, and there is no need to sell stock when the total
capital required is only $200,000.
67
Answer (A) is correct. The three elements required to calculate the cost of equity capital are (1) the dividends per share,
(2) the expected growth rate, and (3) the market price of the stock. Because growth is not expected, the calculation
is simply to divide the dividend of $7 by the $100 market price of the stock to arrive at a cost of equity capital of
7%.
Answer (B) is incorrect because 7.6% would be the cost of new equity capital from selling stock if the stock resulted
in proceeds of $92 per share. Answer (C) is incorrect because 7.4% would be correct only if new stock were sold
and resulted in proceeds of $95 per share. Answer (D) is incorrect because dividing the $7 dividend by the $100
market price of the stock produces a ratio of 7%, not 8.1%.

66
Answer (B) is correct. According to the dividend growth model, the three elements required to calculate the cost
of equity capital are (1) the dividends per share, (2) the expected growth rate, and (3) the market price of the stock.
If flotation costs are incurred when issuing new stock, they are deducted from the market price to arrive at the
amount of capital the corporation will actually receive. Accordingly, the $100 selling price is reduced by the $3
discount and the $5 flotation costs to arrive at the $92 to be received for the stock. Because the dividend is not
expected to increase in future years, no growth factor is included in the calculation. Thus, the cost of the common
stock is 7.6% ($7 dividend ÷ $92).
Answer (A) is incorrect because the 7.0% figure would only be correct if $100 were received from the sale of the
stock. Answer (C) is incorrect because 7.4% would be correct only if the stock could be sold without giving a $3
discount. Answer (D) is incorrect because 8.1% would be correct only if the amount received were about $86 or
if some growth factor were assumed.
65
Solution:
x*.80-x*.1-.6(.03(10M+x))=2,000,000
x*.80-x*.1-.6(300,000+.03x)=2,000,000
x*.80-x*.1-180,000-.018x=2,000,000
.8x-.1x-.018x=2,000,000+180,000
.682x=2,180,000
x= 3,196,480/10M = 31.96% or 32%

62
Time line:
0 k = 14% 1 2 3 Years
| | | |
-42,000 14,280 16,200 11,400
TV = 18,120
29,520

Numerical solution:
$14,280 $16,200 $29,520
NPV = -$42,000    = $2,916.85 ≈ $2,917.
1.14 (1.14)2 (1.14)3
61
Additional Year 3 cash flows:
3
Salvage value $25,000
Tax on Salvage value (8,880)*
Recovery of NOWC 2,000
Total terminal year CF $18,120

*(Market value - Book value)(Tax rate) ($25,000 - $2,800)(0.40) = $8,880.


60
Depreciation schedule:
MACRS
Depreciation Depreciable Annual
Year Rates Basis Depreciation
1 0.33 $40,000 $13,200
2 0.45 40,000 18,000
3 0.15 40,000 6,000
4 0.07 40,000 2,800
$40,000

Operating cash flows:


1 2 3
1) Increase in revenues $20,000 $20,000 $20,000
2) Increase in costs (5,000) (5,000) (5,000)
3) Before-tax change in earnings $15,000 $15,000 $15,000
4) After-tax change in
earnings (line 3  0.60) $ 9,000 $ 9,000 $ 9,000
5) Depreciation 13,200 18,000 6,000
6) Deprec. tax savings
(line 5  0.40) 5,280 7,200 2,400
7) Net operating CFs
(lines 4 + 6) $14,280 $16,200 $11,400
59
Initial investment:
Cost ($40,000)
Change in NOWC (2,000)
($42,000)
53
$921,250= $250,000 F
F = 3.685
Chart criteria for six years is 3.685 = 16%
52

- $32,000 - $160,000 + $16,000= $(176,000)


Yr 1 = $72,000 x 0.833= 59,976
Yr 2 = $72,000 x 0.694= 49,968
Yr 3 = $72,000 x 0.579= 41,688
Yr 4 = $72,000 x 0.482= 34,704
$ 10,336

35
Machine A = ($40,000 / $300,000) =13.3%
Machine B = ($50,000 / $250,000) = 20%
Machine C = ($75,000 / $500,000) = 15%

You might also like